GENERAL PRACTITIONER EXAM
0 of 100 questions completed
Questions:
Hi, Welcome to General Practioner Exam
You have already completed the Exam before. Hence you can not start it again.
Exam is loading...
You must sign in or sign up to start the Exam.
You have to finish following Exam, to start this Exam:
0 of 100 questions answered correctly
Your time:
Time has elapsed
You have reached 0 of 0 points, (0)
Average score |
|
Your score |
|
Pos. | Name | Entered on | Points | Result |
---|---|---|---|---|
Table is loading | ||||
No data available | ||||
A 32-year-old male presents with episodic jaundice and anaemia and has been diagnosed with glucose-6-phosphate dehydrogenase (G6PD) deficiency. On further testing his wife has normal plasma G6PD activity. What is the risk of their children developing this condition?
Which one of the following statements is correct?
G6PD is X linked- therefore females are carriers, and are not usually affected, unless there is inactivation of their X chromosome, Males are affected.
Males will pass on the ´bad´ X chromosome to their daughters so that they become carriers, and are said above they are not usually affected.
Males pass on their Y chromosomes to any sons; therefore they will not be affected. In the question the male is affected, but as the female has normal levels of the enzyme, we are assuming she is not a carrier (although strictly she could be a carrier and have normal levels). The male will pass on the X chromosome to any daughters, who will not be affected, as they will have a ´good´ X from the mothers, and the father will pass on the Y chromosome to his sons, who will not be affected.
In X-linked recessive inheritance, which of the following is true
Over 250 X-linked recessive disorders have been described.
The commonest include:
The abnormal gene is carried on the X chromosome, and in the carrier female, the normal allele on her other X chromosome protects her from the disease. Since the male does not have this protection, he manifests the disease.
In X-linked inheritance therefore:
The family history may be negative, however, since new mutations are fairly common. Carrier females can be identified from time to time from mild clinical manifestations and from specific tests such as biochemical markers, for example, creatine kinase (CK) in Duchenne muscular dystrophy (DMD).
Interstitial lung disease is characterized by all of the following EXCEPT
Interstitial lung diseases results from attempted repair of an inflammatory lung injury. The earliest detectable lesion is inflammatory exudates of the lower respiratory tract. Epithelial cell injury, possibly caused by oxidants, proteases, or inflammatory cells, is a hallmark of the disease. This results in loss of the epithelial barrier, with transudation of plasma and formation of a fibrin-rich exudate. The resulting intra-alveolar exudate contains an additional group of cytokines and inflammatory mediators. If this exudate is not cleared, it is invaded by fibroblasts and other cells, ultimately resulting in a fibrous scar. As a result of alveolar collapse and intraluminal fibrosis, alveolar surface area is lost.
After an accident, a 24-year-old exhibits left-sided deficits in motor, position, and vibration senses and no longer possesses temperature and pain sensations contralateral to the lesion. Which of the following represents the most likely diagnosis? |
Brown-Sequard syndrome is hemisection of the spinal cord. It is most commonly caused by penetrating injuries, such as those that would occur from a knife wound and many motor vehicle accidents. The manifestations of a hemisected spinal cord include ipsilateral vibration, position, and motor deficits to the lesion, with contralateral deficits in temperature and pain. None of the other answer choices are consistent with the clinical findings. In Amyotrophic Lateral Sclerosis, sensory function generally is spared, as is autonomic and oculomotor activity. Motor function is affected. It is not traumatic.
The most common cancer in American males is
Although lung cancer causes the most cancer deaths in American males, prostate cancer is the most common. The frequency of prostate cancer is followed by lung cancer (A) and colorectal cancer (C), respectively
A person presents with headache, fever, and menigismus associated with a CSF pleoscytosis. Gram stain and cultures are negative and viral meningitis is diagnosed. True statement regarding viral meningitis is
Viral meningitis is usually benign and self-limiting, unlike bacterial meningitis which can be life threatening. Contacts of patients with bacterial meningitis, not viral meningitis, need antibiotic therapy. Antibiotics do not play a role in the management of viral meningitis.
FALSE statement regarding persons with allergic rhinitis is which one of the following?
Persons with allergic diatheses respond to an inhalant allergen by producing high levels of allergen specific IgE. This IgE response seems to be controlled by immune response genes located within the major histocompatibility complex on chromosome 6. Sensitization to the allergen is necessary to elicit the lgE response. After IgE antibodies specific for an allergen are synthesized and secreted, they bind to mast cells and basophils. Once allergic reactions begin, mast cells amplify them by releasing vasoactive agents and cytokines. Sneezing correlates with the appearance of measurable histamine and PGD2 in nasal washes.
A male aged 58 years is diagnosed with Wegener´s granulomatosis. LEAST likely finding in this patient is which one of the following?
Wegener´s granulomatosis is an immune disorder associated with abnormal production of c-ANCA autoantibodies. The vasculitis typically affects small and medium sized vessels. However, there is often an accompanying necrotizing glomerulonephritis and necrotizing granuloma formation in the upper respiratory tract. The therapy for Wegener´s granulomatosis is immunosuppression. “Palpable purpura” is associated with hypersensitivity angiitis.
A child aged 1 year develops voluminous watery diarrhea and vomiting. She is brought to the pediatrician by her parents and evaluated, then sent home with instructions for the parents to give the child an electrolyte replacement solution. The most likely causative agent is best described by which one of the following?
Rotavirus is the major cause of diarrhea in infants and children under the age of 2. The virus replicates in the intestinal mucosa, producing a profuse, watery, non-bloody diarrhea, often coupled with nausea and vomiting. Transmission is by the fecal oral route. The virus belongs to the reovirus family which is the only family of double-stranded RNA viruses. They are naked capsid viruses have extremely segmented genomes and are icosahedral and double shelled. Double-stranded DNA, enveloped complex virus describes the poxvirus family. Double stranded DNA, enveloped icosahedral virus describes hepadnavirus and herpesvirus families. Double stranded DNA, naked, icosahedral virus describes papillomaviruses, polyomaviruses, and adenoviruses. Single stranded DNA, naked icosahedral virus describes the parvoviruses.
An exchange student from panama develops an extremely high fever and becomes delirious. Once the admitting physicians reduce her temperature, the patient is able to inform them that she has had malaria before and that her present symptoms remind her of a relapse. A blood film is positive for large ameboid trophozoites. Stage of this parasite that causes the potential for relapse is which one of the following?
The description of large ameboid trophozoites tells us this patient is infected with Plasmodium vivax malaria. P. vivax and P. ovale are both capable of causing relapse following recovery from the primary infection because both organisms leave dormant forms in the liver, called hypnozoites. P. falciparum and P. malariae do not possess these forms and thus are incapable of causing relapses, but if drug treatment of the erythrocytic stages is inadequate, the symptoms can “recrudesce” due to renewed multiplication in the bloodstream. Relapses and recrudescences look the same symptomatically because once parasites begin multiplying again in the bloodstream, the cyclic fevers return. This distinction is medically important, because relapses can be prevented by destruction of hypnozoites with primaquine, by so called “radical cure.” Radical cure is not necessary following Infection with the two species of Plasmodium that do not produce hypnozoites. Gametocytes of P. falciparum and P. vivax are the haploid forms of the malaria parasite, which must be ingested by mosquitoes in a blood meal so that fertilization can occur in the mosquito stomach and a new generation of infectious sporozoites can be produced. Gametocytes die if they are not taken up by mosquitoes, and they play no role in the human with reinitiating symptoms. Hypnozoite of P. falciparum is not a possible answer since neither P. falciparum nor P. malariae leaves these dormant stages in the liver. Merozoites of P. falciparum and P. vivax are parasite daughter cells. Exoerythrocytic merozoites are produced by the process of schizogony (asexual fission) in the liver, and erythrocytic merozoites are produced by schizogony in erythrocytes. Although the release of merozoites from red blood cells is associated with the release of pyrogens and a spike of fever these forms are not responsible for the potential for relapse.
A 55 year old HIV+ male is hospitalized with Pneumocystis jiroveci pneumonia. His CD4 cell count is 30/mm^3. A Western blot is positive for antibodies to gp41 and gp120, but negative for antibodies to p24, p17, and p7p9. The reason for production of antibodies to envelope antigens until the death of the patient, while disappearance to antibodies to internal structural proteins is
During full blown AIDS, the patient has arrived at a point at which he no longer has enough helper T lymphocytes to promote the production of the advanced immunoglobulins (IgG, IgA, IgE). Production of IgM is possible with out T-cell help, and continues against the envelope glycoproteins because those antigens are constantly being altered by the process of random mutations (genetic drift). The patient is left in the final stages of the disease, making constant, repetitive primary immune responses to the changing envelope antigens, while IgG antibodies against the structural proteins of the virus stop being made. Thus, most patients will revert to being seronegative on ELISA, which primarily tests for IgG antibodies against the p24 capsid protein. Paradoxically, total immunoglobulin levels during full-blown AIDS may rise, reflective of the fact that B cells, in the absence of T cell help, are “stuck” making IgM in huge amounts. The statement that envelope glycoproteins of the virus are heavily glycosylated is true, but it does not explain why antibodies to the structural proteins fail while antibodies to the envelope antigens rise during full blown AIDS. The heavy glycosylation of HIV antigens tends to “hide” them from immunologic recognition, so if this were the only factor, concentrations of all antibodies would fall in full-blown AIDS. It is not true that once the population of susceptible cells falls to a certain level, viral structural proteins are no longer produced. All virion antigens are produced throughout infection, until the death of the patient. The expression of some antigens in the bloodstream (e.g., p24) is limited in most of the infection by the production of anti-p24 antibodies that will remain in the patient until progression to full-blown AIDS. The statement that the virus is parasitic in the chromosomes and hidden from the immune response is partially true. The retroviridae are unique in that they parasitize the chromosomes of the host, but immune responses are still made against cell surface-expressed antigens. It is not true that the virus overproduces envelope antigens. Neisseria meningitidis is a bacterium known for overproduction of outer membrane proteins, but HIV does not do anything analogous.
A 3 day old infant has a localized, crusted infection near the umbilical stump. In the next 24 hours tender reddened patches appear near the crusted area and then spread to produce painful and generalized skin erythema. By the next day, large flaccid blisters appear that easily rupture. In areas where the epidermis has been rubbed by the infant´s bed, large sheets of epidermis have peeled off. He appears very ill and temperature instability and chills. The most likely diagnosis is
This infant has staphylococcal scalded skin syndrome, which is a severe skin condition with acute widespread erythema and epidermal peeling as a result of staphylococcal exotoxin. Susceptible populations include infants young children, and adults who are immunosuppressed or have renal failure. The history given here is typical for the presentation in babies. Skin biopsy (or examination of exfoliated skin) with frozen-section analysis can provide rapid diagnosis; cultures should also be obtained from the skin and nasopharynx. Prompt therapy with antistaphylococcal antibiotics will usually prevent death. These babies should be treated as if they had been burned. Special care should be taken to prevent accidental nosocomial spread of the infection to other babies in the unit. Survivors usually have no permanent sequelae. Impetigo is a staphylococcal skin infection that does not produce large blisters. Scarlet fever can also produce severe erythema but has a peak incidence in; children from 4 to 8 years of age. Toxic epidermal necrolysis can appear clinically similar to staphylococcal scalded-skin syndrome, but it occurs in older patients, often in association with a new medication. Toxic shock syndrome can also produce severe erythema but typically occurs in teenagers or adults.
A neonate is noted to have numerous light brown macules dispersed across her skin. The significance of this feature is due to its strong association with the development of which one of the following tumor?
Congenital “cafe-au-lait” spots are present in more than 90% of patients with neurofibromatosis. This autosomal-dominant disorder is characterized by multiple neural tumors, especially neurofibromas, pigmented hamartomas of the iris (Lisch nodules), and cafe au-lait spots, which usually occur over nerve trunks. Although the majority of neurofibromas in this disease are benign, the tumors can be quite disfiguring and psychologically damaging. Basal cell carcinoma (BCC) is a common slow-growing tumor of sun exposed skin. It develops in adulthood after years of chronic sun damage. Although malignant melanoma may arise from large congenital nevi, BCC does not. Neuroblastoma is a common childhood tumor that arises anywhere along the sympathetic chain, and most commonly in the adrenal medulla. The tumor usually presents as an abdominal mass and is not associated with skin findings. The ocular neoplasm associated with neurofibromatosis is the Lisch nodule, not the retinoblastoma. Retinoblastoma is a neuroepithelial tumor usually identified by funduscopic examination of a child with visual changes. Wilms tumor is a neoplasm of primitive renal blastema that may be associated with congenital malformations of visceral organs, notably the adrenals and gonads, but not skin. It also presents as an abdominal mass, and is highly survivable.
A 67 year old man suffers MI due to thrombotic occlusion at the origin of the left circumflex artery. Cardiac catheterization reveals that he has a left dominant coronary circulation. Which areas of the heart have likely suffered ischemic necrosis?
A right dominant coronary circulation is present when the posterior descending branch originates from the right coronary artery (80% of individuals). On the contrary the posterior descending artery originates from the left circumflex artery in a left, dominant circulation (20% of individuals). The posterior descending branch gives blood to the posterior half of the interventricular septum. Occlusion of the left circumflex artery in a left dominant circulation will therefore lead to ischemic necrosis in the left ventricular wall and the posterior interventricular septum. The apex of the left ventricle is dependent on the anterior descending branch; thus, occlusion of the left circumflex does not affect this portion of the left ventricle. Infarction of the lateral (free) wall alone will result from occlusion of the circumflex in a right dominant circulation. An isolated infarction of the posterior interventricular septum arises from occlusion of the posterior descending branch. Isolated infarctions of the right ventricular wall are very are and would be caused by occlusion of branches of the right coronary artery.
According to the husband of a 39 year old obese female she is always tired during the day because she is a “major snorer” and is frequently startled from sleep by choking. Her BP is 150/90 mm Hg, and pulse is 75/mm. Examination is unremarkable. Which condition is likely to be present during the nocturnal events described?
This patient most likely has obstructive sleep apnea, which is a transient airway occlusion that causes apnea (periodic cessation of breathing), progressive asphyxia, and arousal from sleep. This occurs many times during the night and leads to increased cardiac after load, pulmonary and systemic vasoconstriction, acute carbon dioxide retention, transient arterial hypoxia, cerebral dysfunction, sleep fragmentation and excessive motor activity. Arterial oxygen and carbon dioxide return to normal between periods of apnea. Options A and B are incorrect because this disorder does not alter the diffusion characteristics of the lungs. It neither increases thickness of the diffusion barrier nor decreases surface area. Hyperpnea is incorrect because ventilation decreases during apnea. Shunt occurs when blood passes through a region of the lung in which there is no ventilation or there is no diffusion. It refers specifically to a regional disorder. This is not the same as decreased ventilation throughout the lungs, which is called hypoventilation; that is what occurs during periods of apnea.
A girl aged 3 years presents with recurrent subcutaneous abscesses and furuncles. She has a history of pneumonia due to Serratia marcescens. Which enzyme is most likely deficient in her neutrophils?
The child most likely has chronic granulomatous disease (CGD), which results in recurrent staphylococcal infections (such as abscesses and furuncles) and an increased incidence of pneumonia due to Serratia marcescens. Organisms that cause recurrent infections in CGD patients are those that are catalase-positive. Neutrophils are unable to generate bacteria killing superoxide radicals because of lack of NADPH oxidase. Patients with CGD are susceptible to chronic infections with catalase-positive organisms. Addolase B cleaves fructose-l-phosphate to glyceraldehyde and dihydroxyacetone. Deficiency of aldolase B can produce hereditary fructose intolerance. Galactokinase phosphorylates galactose prior to the sugar´s entry into the glycolytic pathway; deficiency of this enzyme can produce galactosemia. Glucokinase and hexokinase catalyze the first reaction in glycolysis. Their deficiency is not usually described, probably because these enzymes are so necessary that a significant deficiency would be incompatible with life. Glucose-6-phosphate dehydrogenase is the enzyme that starts the hexose monophosphate shunt; its deficiency causes a hemolytic anemia.
A 54 year old nonsmoker and non-alcoholic woman presents with fatigue. She has been weak and tired for the past few months and has lately been feeling palpitations. CBC shows a normochromic anemia with no reticulocytes. RBC morphology is normal. Platelets and myeloid cells are unaffected. Bone marrow biopsy is normocellular and is significant for a lack of erythroid precursors, but all other elements are normal. She should be further evaluated for which condition?
In the rare pure red cell aplasia, the erythroid marrow elements are absent or nearly absent, while granulopoiesis and thrombopoiesis remain unaltered. This condition occurs in both primary and secondary forms, both of which are thought to be related to autoimmune destruction of erythroid precursors. There is a relatively weak general association between cancers and red cell aphasia, which is probably due to triggering of autoimmune disease by the cancers. In addition, be aware that there is a specific association between thymic tumors (thymoma) and autoimmune hematologic diseases, specifically pure red cell aplasia. This association is strong enough that a patient with pure red cell aplasia should be specifically evaluated for thymoma. The other answer choices are distracters with no specific association with red cell aplasia, although cancer in general can be a predisposing factor.
A 44 years old man has fatigue and increased frequency of urination. Further history reveals a 5´10″ 240 lb. individual whose fat is centrally distributed. Urine is positive for glucose, but negative for ketones. His blood sample is likely to show which of the following compared to that of normal individual?
A boxer “knocked out” by a blow to the lateral skull during a match recovers after a few minutes, and is asymptomatic for the next 12 hours. He then develops severe headache and is brought to the ER. During evaluation, he does not remember his last name and believes he is at home. He also has nausea and vomiting. The most likely diagnosis is
All of the lesions listed in the answer choices can occur in brain trauma. However, the scenario described is classic for epidural hematoma. In this scenario, a severe blow to the lateral skull causes both skull fracture and laceration of the middle meningeal artery (know the name of this artery as it is frequently tested), leading to a momentary loss of consciousness, which is followed by a lucid (asymptomatic) period of 1-18 hours before the patient´s neurologic condition deteriorates. This scenario is frequently tested on examinations, but you should be aware that in real clinical life, there may be no initial loss of consciousness and the patient may not have sought medical help for skull trauma. A CT scan of the head will typically demonstrate a lens shaped (biconvex) hematoma. Basilar skull fractures are usually located in the vicinity of the petrous bone or along the sphenoid bone. Clinical signs of basilar skull fracture include hemotympanum (blood visible behind the tympanic membrane), delayed ecchymosis over the mastoid process (Battle´s sign), or periorbital ecchymosis (“raccoon sign”). CSF leakage (sometimes from the nose), or pneumocephalus may also occur. Intracerebral hemorrhage is most often due to hypertension, anticoagulant use, cerebral amyloid angiopathy, or cocaine and/or methamphetamine abuse. Subarachnoid hemorrhage is most commonly caused by rupture of a cerebral berry aneurysm, but may also be associated, with arteriovenous malformations or intraparenchymal hemorrhage. Whenever a person complains of the worst headache of my life, consider subarachnoid hemorrhage as a possible cause. Subdural hematomas are due to rupture of the bridging veins between the periosteal dura and the superficial cerebral veins. Blood accumulates below the dura, producing symptoms of increased intracranial pressure in minutes to hours (acute subdural hematomas), or after weeks or months (chronic subdurals, often seen in the elderly).An acute subdural may present like an epidural hemorrhage, but a forceful blow to the lateral skull producing skull fracture is generally associated with an epidural, rather than a subdural, bleed. On head CT, subdural hematomas classically present as crescent shaped hematomas
A 75 year old healthy woman injured her right leg 2 days ago and has been bedridden since the accident. She became delirious 2 hours ago. Her temperature is 99°F, BP is 120/70mm Hg, heart rate is 110/min, and respiratory rate is 32/min. Pulse oximetry shows an oxygen saturation is 80%. CXR is normal. The most likely diagnosis is
Hip fracture and prolonged bed rest are classic risk factors for the development of pulmonary thromboemboli (PE). Common clinical manifestations of PE are hypoxia (due to ventilation/perfusion mismatch) despite a normal chest X-ray, tachycardia, and delirium in older patients. Cerebral hemorrhage might cause delirium but would not directly cause hypoxia unless the patient was hypoventilating (e.g. because of brainstem involvement). Cerebral infarction could produce delirium but would not directly cause hypoxia unless the patient was hypoventilating.
Myocardial infarction could account for delirium and tachycardia, but not for hypoxia with a normal chest X-ray. Severe congestive heart failure after myocardial infarction could cause hypoxia due to pulmonary edema, but the chest X-ray would not be normal. A pulmonary infection may cause delirium, tachycardia, and hypoxia, but the onset of symptoms would likely be slower, the chest x-ray would be abnormal, and the patient would be likely to have a higher fever.
According to the wife of a 47 year old hypertensive male he had taken a nap but she could not wake him up. His past medical history is significant for renal failure. The patient dies in the emergency department. At autopsy kidneys showed numerous cystic dilations. In which location did hemorrhage likely occur?
This patient had adult polycystic kidney disease (APKD) a hereditary disease in which the kidneys become filled with spherical cystic dilations. The disease typically presents with renal failure in the 40s to 30s and is characterized by marked renal enlargement due to numerous fluid-filled cysts. APKD is highly associated with hepatic cysts and berry aneurysms in the circle of Willis that may rupture, producing spontaneous subarachnoid hemorrhage. Infrequently, APKD also produces cysts in the pancreas, spleen, or lungs, but these are not clinically relevant. Hypertensive intracranial bleeds can occur in the basal ganglia; however, the patient´s blood pressure is well controlled. Epidural hemorrhage is commonly seen following trauma to the skull. Classically, the artery damaged is the middle meningeal artery. Intraparenchymal bleeds may be secondary to such things as emboli, metastasis, trauma, or hypertension. Subdural bleeds are from rupture of the bridging veins rather than aneurysmal rupture.
A 43 year old male previously diagnosed with kidney stones has gnawing, burning epigastric pain. He also notes moderate to severe diarrhea. Measurement of his basal gastric acid output reveals that it is markedly elevated. His symptoms are likely the result of which neoplastic syndrome?
Pancreatic islet tumors, which may produce gastrin and secondarily Zollinger Ellison syndrome, are a feature of multiple endocrine neoplasia type I (MEN I). Other features of MEN I are tumors of the parathyroid (the resulting hypercalcemia and hypercalciuria leads to kidney stones), adrenal cortex, and pituitary gland. In general, the tumors in the MEN syndromes may be expressed at different times in a patient´s life, and not all patients exhibit the full syndromes. Endocrine tumors are not a feature of familial polyposis coli, which is instead characterized by colonic polyps and colon cancer. MEN type IIA features tumors of the adrenal medulla (pheochromocytoma), medullary carcinoma of the thyroid, and parathyroid hyperplasia or adenoma. MEN type IIB closely resembles type IIA but also includes mucosal neuromas, and less often includes parathyroid diseases. MEN type III is now considered to be the same as MEN type IIB.
A 32 year old female is diagnosed with breast cancer. Her mother had cervical cancer at age 35, her grandfather had colon cancer at age 36, and her brother was recently diagnosed with lung cancer. If genetic analysis is performed, what would her genotype at the p53 tumor suppressor gene locus?
Typically; cancers are precipitated by a number of “hits” or mutation/deletion events that endow the transformed cell with a survival and/or growth advantage such that it is no longer restricted by the normal cellular checks and balances. The mutation events commonly include (1) loss of function of both copies of a tumor suppressor gene; (2) mutation of a proto-oncogene; or (3) fusion of a proto-oncogene with a strong promoter/enhancer. p53 is a tumor suppressor gene that, when mutated or deleted, seems to play a role in many different types of cancers. If an individual inherits a mutated allele of a tumor suppressor, the chance of acquiring a second mutation in the other allele is extremely high. This second mutation “knocks-out” the normal tumor suppressor function, giving the cell a growth/survival advantage that can lead to the cancer phenotype. Members of families that are strongly predisposed to many different types of cancer may have an inherited mutant allele of p53, referred to as Li-Fraumeni syndrome. Note that although the cells of the tumor may have lost both copies of the tumor suppressor gene, analysis of the blood sample would reveal only the one mutant or deleted allele in other cells. In a normal blood sample, one would not find deletions in both alleles of p53. Loss of both copies of p53 is not compatible with normal cellular function. Normally, being diploid, individuals have two wild-type copies of each allele. Amplification of certain proto-oncogenes (e.g., neu in breast cancers or N-myc in neuroblastomas) is associated with a poor prognosis, but an extra copy of a tumor suppressor gene would probably not contribute to the early development of cancer. Some cancers are precipitated by the fusion of a proto-oncogene with a second gene or with a new promoter/enhancer region, leading to the unregulated overexpression of the proto-oncogene (e.g., the Philadelphia chromosome in chronic myelogenous leukemia [CML]). In contrast to option E, mutations in tumor suppressor genes that produce cancer are generally “loss of function” mutations resulting from deletions, frameshift mutations, or nonsense mutations.
A chronic alcoholic male presents with confusion, ataxia, and nystagmus. Administration of thiamine leads to rapid symptomatic improvement. Neuropathologic change that accounts for this presentation is
The clinical picture of confusion, ataxia, and variable disturbances of eye movements (e.g., nystagmus and ophthahnoplegia) is due to Wernicke encephalopathy, which results from thiamine deficiency. Hemorrhage and necrosis in the above mentioned areas are the underlying pathologic changes. Alcoholics are particularly prone to this condition. Axonal degeneration of posterior and lateral columns of spinal cord, or subacute combined degeneration, results from two different etiologies: vitamin B12 deficiency and HIV myelopathy. Ataxia, numbness, and spastic paresis of the lower extremities are the clinical manifestations. Demyelination affecting the central region of the basis pontis is known as central pontine myelinolysis. This complication is most commonly encountered in severely malnourished and dehydrated alcoholics. Rapid correction of hyponatremia triggers this condition. Demyelination involving the corpus callosum and anterior commissure, or Marchiafava Bignami disease, is a rare disorder associated with chronic alcoholism. It is probably related to some dietary deficiency. Loss of neurons in the neocortex, cerebellar Purkinje cells, and hippocampal pyramidal neurons results from hypoxia or hypoglycemia related necrosis. These neurons are the most vulnerable to hypoxic or hypoglycemic injury.
A male infant begins to have persistent, projectile, non-bilious vomiting at age 2-3 weeks. Condition that is likely responsible for these symptoms is which one of the following?
Congenital hypertrophic pyloric stenosis typically presents as described in the question stem and is more frequent in male infants. Physical examination reveals a palpable ovoid mass in the epigastrium; surgical splitting of the hypertrophic pyloric muscle is curative. The etiology is not well understood. Caroli disease is a malformation that causes segmental dilatation of the intrahepatic biliary tree. Vomiting is not a prominent feature. Cystic fibrosis presents with malabsorption and/or pulmonary infection due to a global defect in chloride secretion. Diaphragmatic hernia is characterized by respiratory insufficiency at birth. Gastric ulcers would be quite unusual in an infant.
A 26 year old female is constantly thirsty and has to urinate every 2 hours. Her plasma osmolality is 295mOsmol/kg, and her urine osmolality is 100mOsmol/kg. Her urine is negative for glucose. She is deprived of fluids for 3 hours as a part of her workup. Her urine osmolality remains 100mOsmol/kg. One hour after injection of arginine vasopressin (AVP), her urine osmolality becomes 400mOsmol/kg. The most likely diagnosis is
ADH (antidiuretic hormone, vasopressin) is normally released from the posterior pituitary in response to hypovolemia and increased osmotic pressure. It acts on the collecting duct of the nephron, increasing its permeability to water. This concentrates the urine and conserves water. Diabetes insipidus (DI) is a condition in which ADH function is absent. Two forms of DI exist: neurogenic and nephrogenic. Neurogenic DI is due to absence of ADH secretion from the posterior pituitary; thus, serum ADH levels are always low, even in states of serious volume depletion. Nephrogenic DI is due to complete or partial resistance to ADH; therefore, even though ADH levels are high, the hormone has no effect on renal water regulation. You are told in the question that the patient´s urine did not concentrate on a water deprivation test. This immediately suggests DL (Normally, when an individual is deprived of fluids for 3 hours, her kidneys respond by concentrating the urine and conserving water). Your task then becomes determining which type of DI the patient has. This is achieved by evaluating the results of the arginine vasopressin (AVP) suppression test. Patients with neurogenic DI readily respond to AVP by producing more concentrated urine. Patients with nephrogenic DI do not concentrate urine in response to AVP because of continued renal resistance. Although thirst is a common symptom of diabetes mellitus, the absence of glucosuria argues against this diagnosis. Primary polydipsia usually occurs in patients taking antipsychotic drugs that cause dry mouth (e.g., phenothiazines), promoting the sensation of thirst. As a result, these patients drink large quantities of water. They excrete dilute urine, and eventually may dilute their serum as well, to the point of electrolyte imbalance. On water deprivation, however, their urine will concentrate normally. SIADH (syndrome of inappropriate antidiuretic hormone) occurs when there is oversecretion of ADH, for example by a small cell carcinoma of the lung or other ADH producing neoplasm. One would expect test results opposite to those in this patient. In contrast to DI, the diagnosis of SIADH is usually made with a water load test.
A 25 year old man has progressive, painless enlargement of neck lymph nodes. CXR followed by CT scan shows marked enlargement of mediastinal nodes. No nodules are seen in the liver or lungs. When evaluating the biopsy of one of the nodes, the pathologist should specifically look for which one of the following?
A 31 year old obese female has a 4 month history of amenorrhoea. She takes no specific therapy. She has two children and her husband has a vasectomy. Exam reveals no abnormality. Serum oestradiol, LH, FSH are decreased. Serum prolactin is 800 mU/L (50-500). Most appropriate next investigation would be
This patient has hypogonadotrophic hypogonadism as evidenced by suppressed LH/FSH and a low oestradiol concentration. This would exclude pregnancy as a cause and polycystic ovarian syndrome is also unlikely. In the presence of a raised prolactin concentration, a microprolactinoma would be the most likely explanation for this patient´s symptoms and results. This may be demonstrated by a pituitary MRI scan. An insulin tolerance test would usually be entirely normal in a microprolactinoma.
Out of the following which one is not a feature of zinc deficiency?
Zinc deficiency is associated with dwarfism, hypogonadism, hepatosplenomegaly, rough and dry skin, mental lethargy and geophagia. Zn supplementation has also been shown to improve neuropsychological function in Chinese children, and Zn deficiency is associated with adverse pregnancy outcomes.
An HIV positive African female aged 32 years presents with a two week of greenish, frothy, itchy vaginal discharge. The most likely cause of her discharge is which one of the following?
Trichomonas vaginalis causes itchy, frothy, greenish vaginal discharge. Candida albicans causes a white, curdy, itchy vaginal discharge. Chlamydia and gonorrhoea do not cause itchy, frothy vaginal discharge and both can be asymptomatic.
Foreign body causes foul smelling vaginal discharge.
Leukotrienes:
Leukotrienes are synthesized by leucocytes. They are mediators of allergic reaction. They increase vascular permeability and attract neutrophils and eosinophils to inflammatory sites. Leukotrienes are synthesised via the lipoxygenase pathway. Leukotrienes D4 has been identified as SRS-A which causes bronchial wall and intestinal smooth muscle contraction (not dilatation). Leukotrienes also stimulate mucus production, an important consideration in the pathophysiology of bronchial asthma.
A 45 year old woman working in a dye factory is diagnosed with cancer. She is most likely at risk of having which of the following cancer?
The use of beta naphythalamine, a substance used in dyes, predisposes individuals to the onset of bladder cancer.
α1-antitrypsin deficiency is NOT explained by which of the following?
α1-antitrypsin is a circulating glycoprotein, which functions to inhibit neutrophil elastase. Of the 16 recognized alleles, only the PiZ and PiS are associated with clinical disease. Pulmonary disease is directly linked to the amount of functional enzyme in the serum. Liver disease results from retention of an abnormal alpha1 antitrypsin within the hepatic endoplasmic reticulum. This occurs primarily with the presence of the Z allele. Persons homozygous for null mutations do not develop liver disease. Liver disease develops independently of lung disease.
Workers in which of the following occupations are NOT at increased risk of lead poisoning?
Which two organs are primarily affected by Goodpasture´s syndrome?
Goodpasture´s syndrome affects the lungs and the kidneys. It causes both pulmonary hemorrhage and acute glomerulonephritis because the body recognizes a self antigen (in this case, a basement membrane protein) which elicits an immune response. The other organs are not typically involved.
A 55 year old man presents with fatigue and weight loss of 30 pounds in the past month. On examination marked splenomegaly is revealed. Bone marrow yields a “dry tap.” Lymphoid cells obtained during this aspiration are positive for tartrate resistant acid phosphatase and CD25 surface antigen. The patient is most likely suffering from?
Hairy cell leukemia is a B-cell lymphoma, which is characterized by tumorigenic production of B-cells, which have projections that are analogous to hairs on at scalp. Hairy cell leukemia cells are TRAP (tartate-resistant acid phosphatase) positive and CD25 positive. Typically, bone marrow aspiration yields a “dry tap” owing to increased marrow reticulin content. Patients are often anemic in hairy cell leukemia, explaining the fatigue of the patient described in the question. None of the other lymphoid disorders present with the same types of laboratory test results
A 52 year old black diabetic male with renal failure presents with a painless, soft tissue mass on his right shoulder. His phosphate level was elevated. Biopsy of the mass revealed fibroblasts and collagen fibers. Right shoulder X-ray revealed calcification of the soft tissue mass. The most likely diagnosis is
Tumoral calcinosis is a periarticular soft tissue mass surrounded by a connective tissue capsule and containing chalky material. This disorder is associated with diabetes, renal failure, and hyperparathyroidism. It is more prevalent in the African American population. Neurofibromatosis (von Recklinghausen´s disease) is characterized by multiple cafe-au-lait spots on the skin and multiple peripheral nerve tumors. It is inherited as an autosomal dominant trait. Soft tissue masses are not visible after bone trauma, although calcifications are seen on X-ray. Pseudogout (chondrocalcinosis) can present as a painful joint attack. Rhomboid crystals are visible after aspiration of the affected joint. This is more prevalent in the elderly. Tuberous xanthomas present as asymptomatic yellowish plaques and are associated with hypercholesterolemia. They are composed of lipid deposits, not fibroblasts.
Which one of the following is NOT included in macrophage secretions?
Macrophages are long lived phagocytic cells important in the inflammatory response. Some of the many secretory products of macrophages include binding proteins such as fibronectin, enzyme inhibitors such as alpha-2 macroglobulins, pyrogens such as interleukin I, and tumor necrosis factor, also known as cachectin. Exotoxin is secreted by bacteria.
The exudate from an abscess is gram stained and cultured on blood agar. Gram stain reveals purple cocci in large clumps. On blood agar, the colonies break down hemoglobin to a colorless product, and when hydrogen peroxide is added, gas bubbles are produced from the surface of the colonies. Fibrous clot is formed when organisms are added to animal serum tubes. This agent protects itself from phagocytosis by which one of the following?
The tests mentioned reveal a gram positive coccus with the characteristics of Staphylococcus (gram positive cocci in grape like clusters). The organism is beta hemolytic (breaks hemoglobin down to a colorless product) and catalase positive (produces water and oxygen gas from the breakdown of hydrogen peroxide). Production of a fibrin clot in a tube of animal serum reveals that the organism is coagulase posltive. The only organism that fits all of these criteria is Staphylococcus aureus. S. aureus evades phagocytosis by producing protein A, which binds to the Fc component of IgG, thus preventing opsonization. Capsules of bacteria and fungi are antiphagocytic, but this is not the means used by S. aureus to evade phagocytosis. Diffuse fimbriate layers of gram positive bacteria cause adherence to eukaryotic cell surfaces. They do not protect against phagocytosis. IgA proteases are produced by Streptococcus pneumoniae, Neisseria meningitidis, N. gonorrhoeae and Haem influenzae. They do not protect against phagocytosis, although they do tend to prevent the bacteria that produce them from immediately being shed from the mucosal surfaces. Inactivation of complement is a mechanism used by Streptococcus pneumoniae to protect itself against opsonization.
A 7 year old child develops edema, hypertension and hematuria several weeks after a severe streptococcal sore throat. Best description of the properties of the most likely infectious organism is given by which one of the following?
The most likely cause of glomerulonephritis following sore throat is Streptococcus pyogenes. This is a Group A streptococcus, which is characterized by beta hemolysis and inhibition by the antibiotic bacitracin. Streptococcus pyogenes pharyngitis can also be followed by rheumatic fever. Streptococcus pneumoniae is an example of an alpha hemolytic streptococcus that is inhibited by optochin and lysed by bile. Streptococcus mutans is an example of an alpha hemolytic streptococcus that is not inhibited by Optochin nor lysed by bile. Enterococcus faecalis is an example of a beta hemolytic streptococcus that can grow in 40% bile and 6.5% sodium chloride. Streptococcus agalactiae is an example of a beta hemolytic streptococcus that is resistant to bacitracin and positive for the CAMP test (an incomplete hemolysin).
Gram stain of the sputum of a child with recurrent upper respiratory tract infections, who has chronic progressive cough, reveals gram negative aerobic rods. The infectious agent for this child´s pneumonia is also causes of which disease?
Gram stain that reveals slightly curved, motile, gram negative aerobic rods is most likely Pseudomonas aeruginosa. Now the question is: which of the diseases listed is also caused by Pseudomonas? The answer is otitis externa. P. aeruginosa is often found in the external ear, especially if the conditions are moist (“swimmer´s ear”) and there is any sort of inflammation. External otitis is usually a benign process, with the only symptom being an itchy, painful ear. If, however, the organism penetrates the epithelium and invades the soft tissue, cartilage, and cortical bone, the process becomes malignant otitis externa, which can progress to osteomyelitis leading to cranial nerve palsies. This condition is most common in diabetics. The treatment is antipseudomonal penicillin and aminoglycosides. Croup, also called laryngotracheobronchitis, is a respiratory disease of children that presents with a characteristic “barking” cough. Croup is caused by parainfluenza virus. Epiglottitis is a potentially fatal infection in children, caused by H. influenzae, which presents with drooling, difficulty breathing, and stridor. The incidence of this disease has dropped dramatically with the introduction of the H. influenzae type B(HiB) vaccine. Meningitis is caused by numerous different bacteria, depending on the age of the patient. The most common causes include S. pneumoniae (elderly people), H. influenzae (unvaccinated children), group B Strep and E. coli (neonates), and N. meningitidis (age 1 month to adult). While P. aeruginosa can cause meningitis, it is nota common cause. The most common causes of otitis media include S. pneumoniae and H. influenzae. Even in cases of external ear infections with P. aeruginosa, the middle ear is typically spared.
A 3 year old mentally retarded is brought for a repair of an ASD. She has mocrocephaly, short palpebral fissures, a smooth philtrum, and a thin upper lip. She also has small teeth and poorly formed ears. She has a limitation in joint movement. Her mother did not receive any prenatal care. Maternal avoidance of which agent would most likely have prevented this condition?
This patient most likely has fetal alcohol syndrome, which is a combination of multiple abnormalities that are caused by maternal intake of alcohol during pregnancy. The craniofacial defects include epicanthal folds, short palpebral fissures, a smooth philtrum, a thin upper lip, small teeth, and poorly formed ears. The patients are often mentally retarded, have limitation of joint movement, and have microcephaly. Major cardiac manifestations include atrial and ventricular septal defects. Pregnant women should not drink any alcohol, because it is not known exactly when and how much alcohol causes this syndrome. Diethylstilbestrol is associated with clear cell adenocarcinoma of the vagina in female offspring of women taking this drug. Folate should be taken by all pregnant women and women planning on becoming pregnant to avoid neural tube defects. This patient´s findings are not caused by maternal intake of folate. Maternal lithium intake is associated with Ebstein anomaly and tricuspid atresia. Other noncardiac teratogenic manifestations are rare. Maternal thalidomide intake is associated with variable cardiac manifestations and phocomelia. It is rarely associated with the craniofacial findings in this patient.
A 28 year old IV drug user has difficulty in swallowing. Oropharynx examination reveals white plaques along the tongue and the oral mucosa. The statement that best describes the appearance of the infectious organism is
This patient has Candida esophagitis. Any time a patient presents with dysphagia or odontophagia, along with white plaques in the oropharynx (thrush), you can assume that the Candida is affecting the esophagus as well. The fact that the patient is an IV drug user makes an opportunistic infection such as Candida more likely. Candida appears as budding yeast with pseudohyphae in vivo. The other answer choices represent the morphology of other important opportunistic fungi: Cryptococcus is an encapsulated yeast. You should think about Cryptococcus neoformans when you´re presented with an immunocompromised patient with neurologic symptoms. The classic clue is the presence of encapsulated organisms observable in an India ink preparation. Mucor and Rhizopus are molds with nonseptate hyphae. You should think about Mucor when you are presented with a diabetic (especially in ketoacidosis) or a leukemic patient with a severe sinus infection. Aspergillus is a mold with septate hyphae. In immunocompromised patients, aspergillosis can present with acute pneumonia, often with cavitation (aspergillomas = fungus balls in the lungs).
A 28 year old woman with a history of pelvic inflammatory disease has severe left lower quadrant crampy pain and spotting, and amenorrhea for the past two cycles. Exam reveals a left adnexal mass with tenderness to palpation. The β-hCG level is elevated. Further evaluation would likely reveal an implantation at which site in the fallopian tube?
This patient has an ectopic (tubal) pregnancy. Patients with a history of pelvic inflammatory disease are more susceptible to this disorder. The ampulla is the most common site of fertilization within the fallopian tube, as well as the most common site for tubal pregnancy. It is the longest region of the tube and has thin walls. The fimbriae of the fallopian tubes are highly unlikely locations for tubal pregnancy. They are mucosal ridges located at the funnel shaped end of the oviduct that are covered with ciliated cells. They beat toward the mouth of the tube, “brushing” the ovum released from the ovary into the fallopian tube. The infundibulum is the technical term for the funnel shaped end of the fallopian tube. It opens to the peritoneal cavity. The isthmus is the narrow, thick walled segment of the fallopian tube nearest to the uterine wall. The uterine (interstitial) segment is the portion of the tube that traverses the uterine wall. Ectopic pregnancies occurring here are at particularly high risk for catastrophic rupture.
A woman aged 55 years has a swollen, red and tender right breast. The clinician palpates a firm area in the breast and suspects inflammatory breast cancer. Which statement best describes the histologic changes observed in this disorder?
Inflammatory breast cancer is a pattern of invasive breast cancer in which the neoplastic cells infiltrate widely through the breast tissue. The cancer involves dermal lymphatics and therefore has a high incidence of systemic metastasis and poor prognosis. If the lymphatics become blocked then the area of skin may develop lymphedema and “peau d´orange” or orange peel appearance. The overlying skin in inflammatory breast cancer is usually swollen, red, and tender. Acute inflammation is a rare finding in breast cancer and may be associated with secondary infection or abscess. Chronic inflammation in breast cancer is a non specific finding. In medullary breast cancer, a type of invasive ductal carcinoma, there are a large number of lymphocytes around the tumor and a desmoplastic reaction is often absent in the surrounding tissue. This type of cancer carries a somewhat better prognosis. Epidermal invasion by cancer cells is a poor prognostic indicator. Intraepidermal malignant cells are called Paget cells. Paget disease of the nipple is a type of ductal carcinoma that arises in large ducts and spreads intraepidermally to the skin of the nipple and areola. There is usually an underlying ductal carcinoma. Fat necrosis is often seen following trauma to the breast, but is not specifically associated with a particular type of breast cancer, although it may be confused with breast cancer if areas of calcification are present.
On examination of the neonate born to a 42 year old woman there is a large defect in the occipital bone through which the posterior portion of the brain has herniated. The most appropriate name for this lesion is
The central neurons system and its overlying bones are subject to a variety of malformations and developmental diseases. The defect described in the question stem is a cranial encephalocele, in which brain herniates through a defect in the skull bones. The most common site for such a herniation is the occipital bone. Small defects in the occipital bone can be treated surgically, but large detects are very problematic, particularly if significant herniation has occurred, since the brain becomes very vulnerable to trauma and infection. Meningocele is the term used when the meninges, but not the brain or spinal cord, herniate through a defect in the bony cranium or spinal column. Myelocele is the term used when the spinal cord herniates through a defect in the spinal column.
Spina bifida refers to vertebral detects through which the spinal cord or meninges may herniate. Syringomyelia is a tubular, fluid-filled cavity within the spinal cord.
BP of a patient who complains of tiredness and muscle weakness is 165/95 mmHg. Plasma Na+ is slightly increased and plasma K+ is decreased. Hematocrit is also low. Plasma renin activity is decreased and serum aldosterone is increased. What is the likely diagnosis?
Conn syndrome, or primary hyperaldosteronism, results from an adrenal tumor that secretes excessive aldosterone. The increased mineralocorticoid effects of aldosterone lead to renal sodium and water retention (which explains the hypertension) and increased renal potassium excretion (hypokalemia). The volume expansion also explains the decrease in hematocrit. The increased blood volume increased blood pressure and hypernatremia will all tend to suppress renin secretion in an attempt to compensate for the increased aldosterone. Addison disease or primary adrenal insufficiency, is characterized by low plasma concentration of aldosterone, hyponatremia, hypotension, and hyperkalemia. In cushing syndrome blood pressure may increased because of crossover mineralocorticoid activity of the increased plasma cortisol. Furthermore cortisol makes blood vessels more responsive to catecholamines, which could increase peripheral resistance. The combination of increased blood pressure and hypokalemia would, if anything, tend to suppress secretion of aldosterone. 21 hydroxylase deficiency is likely to produce hypotension. In the salt wasting variant of this disorder, the plasma concentration of aldosterone is decreased and hyponatremia and hyperkalemia result.
Pheochromocytoma is another endocrine cause of hypertension. The increased plasma concentration of catecholamines can cause increased cardiac output and increased peripheral resistance. Plasma renin activity may be increased because of increased beta receptor activation on juxtaglomerular cells. This could produce increased aldosterone secretion and subsequent salt retention.
A 47 year old chronic alcoholic has nausea, vomiting and epigastric pain for 1 day radiating to his back. He gradually recovered from this acute episode. Following similar episodes over the next 5 years he has gradual onset of weight loss, mid-abdominal pain radiating to his back, and bulky, “greasy stools”. He also has bleeding that is “difficult to stop” after brushing his teeth. What is the likely cause of his condition? |
Severe epigastric pain radiating to the back and accompanied by nausea and vomiting suggests pancreatitis. Pancreatitis is most likely to be encountered in alcoholics (such as this main) and patients with biliary tract disease. This patient has a history of recurrent alcoholic pancreatitis. The development of gradual weight loss chronic pain radiating to the back, and steatorrhea (greasy bulky stools) suggests that he has now developed chronic pancreatitis. This condition is often complicated by both endocrine (diabetes mellitus) and exocrine insufficiency of the pancreas secondary to loss of much of the tissue of the pancreas to the disease process. The absorption of lipid soluble vitamins (vitamins A, D, E and K) is indirectly affected by this condition because these micronutrients need normal pancreatic and intestinal function in order to be absorbed. This patient´s bleeding is most likely secondary to a deficiency of vitamin K, which is essential for the proper functioning of the coagulation system.
Pancreatic adenocarcinoma arises from the ductal epithelium. It is most common in people between ages 60 and 80 and presents with vague signs and symptoms until late in its course, when abdominal pain, obstructive jaundice, and other manifestations such as migratory thrombophlebitis occur. Cholangiocarcinoma is a neoplasm of intrahepatic bile ducts that typically produces jaundice (because of biliary tract obstruction). It has a very poor prognosis, and usually occurs in patients with a prior history of primary sclerosing cholangitis. Primary sclerosis cholangitis is characterized by fibrosis of the intrahepatic and extrahepatic bile ducts. It is most common in men (age 20 to 40 years) the majority is associated with ulcerative colitis and the typical presentation is obstructive jaundice. Gastric outlet obstruction would present with postprandial vomiting and early satiety. It can be caused by benign causes such as peptic ulcer disease and gastric polyps, or by neoplasias such as pancreatic or gastric cancer.
A man has constant, severe right upper quadrant pain that radiates to the right shoulder. He is febrile has diffuse tenderness to palpation in all four quadrants, with the right upper quadrant markedly more severe. Deep inspiration while palpating the area below the right costal margin cause marked pain. Leukocytosis and mild total bilirubin elevation are present. Which structure is likely involved in producing his symptoms?
This patient has a history consistent with acute cholecystitis. Patients are febrile, with unrelenting RUQ pain and tenderness. In acute cholecystitis, a biliary stone becomes lodged in the cystic duct, causing an occlusion. Inflammation/infection of the gall bladder results. Leukocytosis is present but only mild elevations in bilirubin are seen. (The flow of bile is not impeded into the small bowel.) The right and left hepatic ducts do not typically become occluded by stones. The right and left hepatic ducts join to form the common hepatic duct which merges with the cystic duct to form the common bile duct. Stones occluding the common bile duct (choledocholithiasis) result in colicky RUQ pain. Patients may have jaundice and a conjugated hyperbilirubinemia if the obstruction persists and the biliary tree becomes infected the life threatening condition cholangitis results. The pancreatic duct does not form stones. Gallstones however may occlude the ampulla of vater, causing pancreatitis.
A 15 year boy with a history of peripheral neuropathy and ocular abnormalities has multiple purplish nodules diffusely over his skin. Other family members have been similarly affected as shown below. It is revealed that he has deficiency of α-galactosidase. Biopsy of skin lesions would most likely reveal that they were
Fabry disease is a lysosomal storage disease with sex linked genetics caused by deficiency of α-galactosidase, which leads to the accumulation of ceramide trihexoside in many tissues (reticuloendothelial, myocardial, ganglion, renal glomeruli and tubules, and connective tissue). Fabry disease is characterized by angiokeratoma corporis diffusum (cavernous hemangioma with epidermal keratosis), thickened blood vessels (with risk of myocardial infarction and stroke), and progressive renal failure and hypertension in adult life. Multiple basal cell carcinomas are seen in familial basal cell nevus syndrome, which is not a lysosomal storage disease. Cystic hygromas are seen in Turner syndrome and are associated with a “webbed” neck. Neurofibromas are seen in neurofibromatosis. Multiple squamous cell carcinomas are seen in skin badly damaged by the sun or radiation, in xeroderma pigmentosa, and in albinism.
A 20 year old boy presents with an enlarged right testicle that was undescended at birth but had self corrected by age 1. Serum AFP is elevated. Biopsy shows multiple mitoses, along with hemorrhage and necrosis. The tumor type is not particularly radiosensitive. The most likely diagnosis is
Embryonal carcinomas occur most commonly in the 20-30 age group and are more aggressive than seminomas. These tumors present with testicular enlargement, and 30% have metastasized at the time of diagnosis. Serum AFP is usually elevated. They are less radiosensitive than seminomas. Interstitial (Leydig) cell tumors can produce androgens, estrogens, or corticosteroids. In children, they often present with masculinization or feminization. In adults, they often present with gynecomastia. Microscopically, tumor cells resemble normal Leydig cells with round nuclei, eosinophilic cytoplasm, and lipid granules. Approximately half the tumors contain cigar-shaped crystalloids of Reinke. Seminomas occur most commonly in the fourth decade of life. Microscopic exam reveals sheets of uniform polyhedral cells divided by fibrous septa of connective tissue; lymphocytes and multinucleated giant cells may also be present. These tumors are highly radiosensitive. Sertoli cell tumors can produce small amounts of androgens or estrogens, but usually not enough to cause endocrinologic changes. They may present with testicular enlargement. Microscopic exam reveals uniform, tall, polyhedral cells with clear cytoplasm, growing in cords resembling spermatic tubules.
A 43 year old male previously diagnosed with kidney stones has gnawing, burning epigastric pain. He also notes moderate to severe diarrhea. Measurement of his basal gastric acid output reveals that it is markedly elevated. His symptoms are likely the result of which neoplastic syndrome?
Pancreatic islet tumors, which may produce gastrin and secondarily Zollinger Ellison syndrome, are a feature of multiple endocrine neoplasia type I (MEN I). Other features of MEN I are tumors of the parathyroid (the resulting hypercalcemia and hypercalciuria leads to kidney stones), adrenal cortex, and pituitary gland. In general, the tumors in the MEN syndromes may be expressed at different times in a patient´s life, and not all patients exhibit the full syndromes. Endocrine tumors are not a feature of familial polyposis coli, which is instead characterized by colonic polyps and colon cancer. MEN type IIA features tumors of the adrenal medulla (pheochromocytoma), medullary carcinoma of the thyroid, and parathyroid hyperplasia or adenoma. MEN type IIB closely resembles type IIA but also includes mucosal neuromas, and less often includes parathyroid diseases. MEN type III is now considered to be the same as MEN type IIB.
In a patient with long-standing congestive heart failure exacerbated by periodic acute pulmonary edema bronchoalveolar lavage is performed. Which finding would be consistent with his history of cardiac failure?
Many different types of structures and cells, normal and pathologic, can be found in bronchial and alveolar cavities. These can be observed in bronchial washings obtained by bronchoalveolar lavage-a procedure in which buffered saline is injected into a peripheral bronchus and then reaspirated. Hemosiderin laden macrophages, or heart failure cells, indicate prior episodes of pulmonary edema or intra alveolar hemorrhage of any cause (e.g., Goodpasture syndrome). Anthracotic macrophages contain black cytoplasmic carbon particles, phagocytosed from polluted air or cigarette smoke. Charcot Leyden crystals are rhomboid structures derived from enzymes present within eosinophils. They are commonly found in patients with allergic conditions (asthma). Curschmann spirals are corkscrew shaped casts resulting from mucus plugs present in smaller bronchi and commonly found in association with chronic obstructive disease. Fat-laden macrophages have been observed in cases of lipid pneumonia.
A 5 year old boy develops large purpuric lesions on the buttocks and extensor surfaces of the arms and legs after an upper respiratory infection. He also has abdominal pain, vomiting, and arthralgias. Urinalysis shows microscopic hematuria. Renal biopsy would probably show which feature?
The presentation in the question stem is classic for Henoch-Schonlein purpura. The distinctive renal biopsy finding is deposition of IgA (together with IgG and complement) in the mesangium of the glomeruli. IgA can also be present in small dermal vessels within the characteristic skin lesions, which consist of subepidermal hemorrhages with necrotizing vasculitis. Vasculitis can also be present in gastrointestinal organs (hence the abdominal symptoms) but is not present in the kidney. The condition typically occurs in young children, between the ages of 3 and 8 years, but it can occur in adults. Adults are more likely to develop severe renal disease with crescentic glomerulonephritis. The clinical course is variable; patients who have more extensive renal damage and proteinuria are more likely to progress to eventual renal failure. Some patients have recurrent hematuria for years. Subepithelial electron dense “humps” are a feature of post streptococcal glomerulonephritis, which can also follow an upper respiratory infection but differs from Henoch-Schonlein purpura by the absence of vasculitis and purpura. Interstitial nephritis and tumor are distracters that are unrelated to Henoch-Schonlein purpura.
A girl aged 18 years whose grandmother was recently diagnosed with breast cancer discovers a large, round, moveable nodule in her left breast. She visits her physician. Which diagnosis should rank highest on the physician´s differential?
Fibroadenoma is the most common benign breast tumor. It occurs in women of reproductive age, generally before age 30, and may be related to increased estrogen sensitivity. It presents as a single, movable breast nodule, not fixed to the skin. Surgical excision is required for definitive diagnosis.
Cystosarcoma phyllodes is a fibroadenoma like tumor that has become large, cystic, and lobulated. It may contain malignant elements. Fibrocystic breast disease is the most common breast disorder. It usually affects women older than 35. It involves a distortion of the normal breast changes associated with the menstrual cycle. Patients often have lumpy, tender breasts, especially during the several days prior to menstruation. Infiltrating ductal carcinoma is the most common type of breast cancer. It occurs most frequently after age 40. Other risk factors include nulliparity, family history, early menarche, late menopause, previous history of breast cancer, obesity, and high fat diet. Intraductal papilloma is associated with bloody or serous nipple discharge. It is most common in women aged 20-50.
A 35 year old man has a 6 month history of tiredness, weight gain and cold intolerance. On examination he appeared hypothyroid and had a firm goitre. Investigations reveal free T4 of 6 pmol/L (10-22) and TSH of 55 mU/L (0.4-5.0).
The most likely diagnosis is
The correct pairing of cell type and substance synthesized by them is which one of the following?
Islet beta cells produce insulin and Amylin, as well as C-peptide, pro-insulin and GABA. Islet D cells produce somatostatin, F cells produce pancreatic polypeptide and alpha cells produce glucagon. Gastric chief cells produce pepsinogen whilst gastric parietal cells produce acid and intrinsic factor.
A female aged 45 years is diagnosed with bacterial endocarditis. The characteristic fundoscopic feature this disease is which one of the following?
Macrocytic anemia is found in all of the following conditions EXCEPT
SLE can be associated with normocytic normochromic anemia (chronic inflammation) or microcytic hypochromic anemia (blood loss). The other entities are associated with macrocytosis. Mild macrocytosis is a common finding associated with rapid blood restoration or production compensating rapid turnover, since in general, “fresh” or newly-produced red cells (reticulocytes) are larger than the mean (average) size, due to slow shrinkage of normal cells over a normal red cell circulating lifetime.
Mycoplasma pneumoniae accounts for up to 20% of all pneumonias in children and young adults. Which of the following is FALSE regarding mycoplasmas?
The Mycoplasmas are the smallest free living organisms. They are bound by a triple layer membrane and have no cell wall. The lack of a cell wall prevents visualization on a Gram stain. The lack of a cell wall further prevents effective therapy with antibiotics which are cell wall active, such as beta lactams and vancomycin. Mycoplasma has a wide range of immunomodulatory effects, including stimulation of T and B lymphocytes, stimulating cytokine production, and producing chemotactic factors. These immunomodulatory effects probably account for the appearance of host autoantibodies. Mycoplasma pneumoniae infection is spread by large droplets of respiratory secretions, particularly in closed environments such as households, dormitories, and institutions. The disease is transmitted during the acute phase of infection. Asymptomatic carriers are inefficient transmitters of the disease. Infection spreads slowly through a population because of the prolonged incubation, which usually lasts two to three weeks.
Varicella zoster virus (VZV) causes two distinct clinical syndromes. True statement concerning infection with VZV is which one of the following?
Varicella zoster virus causes two distinct clinical syndromes-varicella (chickenpox) and herpes zoster (shingles). The disease is highly contagious, with an attack rate of at least 90% in susceptible persons. Humans are the only reservoir for VZV. Transmission is via the respiratory route, with viral replication occurring in the upper respiratory epithelium.Varicella is contagious approximately 48 hours prior to the onset of the rash and for approximately five days following the appearance of the first lesion. Once all lesions have crusted, there is no risk of transmitting the infection. Following clinical recovery from varicella, the virus is thought to remain latent in the dorsal horn cells. Reactivation of the virus presents as herpes Zoster.
An infant aged 2 months is brought because of a fractured arm. Since birth this is his sixth fracture. On examination he has a soft skull that when palpated, gives the impression of loose bones under the scalp. His sclerae are blue-tinged and he does not respond to sound. What is the likely cause of his problem?
The patient has osteogenesis imperfecta (“brittle bone disease”), which is a genetic disease that causes very fragile bones. The type illustrated in the question stem is the severe, lethal form; milder forms also exist. The blue sclera may or may not be present. A variety of mutations appear to cause this clinical syndrome, most of which appear to affect type I collagen and the conversion of calcified cartilage to bone. The severe form is often fatal in infancy.
Marfan syndrome causes tall stature, ocular changes, and aortic dissection, and is not usually diagnosed in infancy. Hurler syndrome causes dwarfism and other skeletal changes, and is not usually diagnosed in infancy. Achondroplasia causes dwarfism. You should be aware that milder cases of osteogenesis imperfecta are sometimes misdiagnosed for years as “child abuse”, which can lead to the emotional trauma of the child being removed from the family.
A male aged 27 years has recurrent episodes of intensely pruritic vesicles symmetrically distributed on his trunk. On electron microscopy, granular deposition of IgA and complement is noted at the dermoepidermal junction. Which underlying condition predisposes patients to the described skin disorder?
Patients with celiac sprue (nontropical sprue or gluten sensitive enteropathy) are prone to the development of dermatitis herpetiformis, the dermatologic diagnosis in this patient. This gastrointestinal malabsorption syndrome is caused by an allergic, immunologic, or toxic reaction to the gliadin component of gluten and has a genetic predisposition. There is reversal of symptoms of celiac sprue with a gluten free diet. Hint: Even if you were unaware of the association between celiac sprue and dermatitis herpetiformis, the mention of IgA and complement as an element in the skin pathology should have increased your suspicion that the associated condition was immunologically related. Lactase deficiency is the most common disaccharidase deficiency (lactase is absent from the brush border of the small intestine). It results in milk intolerance, with symptoms of bloating, diarrhea, and cramping following ingestion of dairy products. Tropical sprue is a disorder of malabsorption of unknown etiology that may be caused by enterotoxigenic Escherichia coli. There is a high incidence in the tropics, where it occasionally occurs in epidemics. Treatment is with tetracycline and folic acid. Ulcerative colitis is associated with a dermatologic condition called pyoderma gangrenosum. This disorder is characterized by blue red ulcerations that surround purulent necrotic bases. Whipple disease is a systemic disorder characterized by clumps of periodic acid-Schiff (PAS)-positive macrophages (full of and surrounded by small bacilli) in the lamina propria of intestines and in mesenteric lymph nodes. It also leads to malabsorption and responds to tetracycline.
An 81 year old hypertensive female is admitted for hip fracture treatment. Her right hip is pinned and she is confined to bed rest. On the third day, she develops chest pain, dyspnea, and cyanosis and dies. What is the most important etiologic factor in her death?
90% of deep vein thrombi are formed in the lower extremities; these thrombi may dislodge and travel to distant sites, most importantly, the lungs. The most frequent cause of pulmonary thromboembolism is deep venous thrombosis of the legs. In this case, the patient´s prolonged bed rest predisposed her to deep venous thrombosis. Other factors that predispose to clot formation include pregnancy, cancer, or other hypercoagulable states. Some emboli are clinically silent others affect only those with preexisting cardiac compromise. However, if the clot is large enough, it will cause nearly instantaneous death. A postoperative patient is at risk of a myocardial infarction; however, in this case, this is not the most likely scenario. A pulmonary embolus can cause heart failure, but this is not the primary etiologic event. Fat emboli can occur after a fracture of long bones; this is also much less likely than a thromboembolus. The fat particles cause a systemic inflammatory response. Veins in the upper limbs rarely develop thrombosis. This occurs only in disorders leading to a marked thrombotic propensity, such as carcinoma associated migratory thrombophlebitis (Trousseau sign), antiphospholipid antibody syndrome, protein C deficiency, and inherited mutations of the factor gene.
An embolus to the pulmonary artery is the typical sequela of deep venous thrombosis, rather than clot formation in the pulmonary vein.
A boy who has had abnormal development of the membranous bones has a broad skull with associated facial and dental anomalies. Which other bones are most likely to also be affected?
In a syndrome called cleidocranial dysostosis, absence of parry of the clavicles accompanies a broad skull, and facial and dental anomalies. Note that you could also have answered this question by noting that of the bones listed, only the clavicles form by intramembranous ossification. The femurs, metatarsals, phalanges, and tibias are cartilaginous (formed by endochondral ossification) rather than membranous bones.
A breast tumor mass of a 54 year old female is removed surgically. The specimen shows negative margins and no lymph node involvement. Immunohistochemical studies are performed to stain for the expression of estrogen receptors, progesterone receptors, and HER2-type EGF receptors. Which marker will likely show immunoreactivity that is limited to the nucleus?
Breast cancer is the second leading cause of cancer deaths among women. While surgical excision with negative margins and no lymph node involvement has a relatively good prognosis for the patient, an adjunct therapy involving either chemotherapy or radiotherapy is strongly recommended for high grade cancers. In particular, tumors that express certain receptors such as estrogen progesterone and the HER2 type epidermal growth factor (EGF) receptors may make the lesion more susceptible to pharmacologic interventions, including the estrogen receptor antagonist tamoxifen and the HER2 EGFR blocker trastuzumab (Herceptin®). Since both estrogen and progesterone receptors are steroid hormone receptors they will mostly localize in the nucleus to act as transcription factors. These would appear as a nuclear pattern of straining. In contrast, EGFR receptors are tyrosine kinase receptors and will be largely localized to the cell membrane surface.
A newborn child born with a congenital infection has excessive growth of new bone on the anterior surface of the tibia with anterior bowing of the bone. This abnormal finding is due to which of the following processes?
The infant has congenital syphilis. The tibial deformity develops when spirochetes localize in the periosteum, where they produce an active inflammation with many plasma cells. This then causes a massive, reactive, periosteal deposition of bone on the medial and anterior surfaces of the tibia (“saber shin”). “Saddle nose” deformity, another feature of congenital syphilis, is due to inflammatory destruction of the nasal and palatal bones. Gummas, in which a central necrotic focus is surrounded by layers of granulomatous and nonspecific chronic inflammation, can also be found in and near the saber shin and saddle nose. The bone changes are not neoplastic, and involvement of the medulla and cortical layer of the bone is less marked than are the periosteal changes.
A 53 year old woman presents with tiredness. There are no specific abnormalities noted on examination. Her T4 level is 21.1 pmol/L (10-22), T3 is 5.2 pmol/L (5-10) and TSH 0.05 mU/L (0.4-5). Thyroid autoantibody titres are all undetectable. The likely diagnosis is
This patient has subclinical hyperthyroidism and, in the absence of the absence of thyroid auto antibodies, the most probable explanation of these thyroid function abnormalities is a solitary toxic nodule.
The level of cellular telomerase activity will affect:
The telomere is a DNA sequence at the end of each chromosome which becomes progressively shorter with each division the cell undergoes. When it is reduced to a critical length the cell is not capable of dividing, the enzyme telomerase is able to lengthen the telomere thus preventing this occurring.
Cysteine proteinases are inhibited by which of the following?
α2 macroglobulin inhibits the cysteine proteinase family of enzymes, while the other molecules do not. Tissue inhibitor of metalloproteinases (TIMP) is a specific inhibitor for the metalloproteinase enzymes. Aprotinin inhibits mast cell tryptase. αl proteinase inhibitor inhibits elastase. αl chymotrypsin inhibits cathepsin G.
All of the following can cause hirsutism, EXCEPT
Estrogen replacement therapy does not cause hirsutism unless the formulation contains androgens. Heritable defects in adrenal steroidogenesis, such as the 21-hydroxylase deficiency result in excess androgen and hirsutism. Polycystic ovarian disease, with its associated elevation of plasma LH and enhanced androgen secretion by ovarian stromal and thecal cells, also results in hirsutism. Adenomas and carcinomas of the ovaries and adrenals, by secreting androgens, result in hirsutism. A variety of drugs, such as minoxidil, diazoxide, and cyclosporine produce hirsutism independently of androgens.
A 35 year old man presents with fever, headache, chills, and malaise 2 weeks after returning from New Delhi, India. He got multiple flea bites on his ankles and calves during his trip. Exam reveals a maculopapular rash on his trunk, arms, and thighs. The most likely cause of his condition is
The symptoms described are characteristic of endemic or murine typhus, caused by Rickettsia typhi. The vector is the rat flea, and the reservoirs are the rat and the rat flea. Unlike other rickettsial infections, R. typhi is frequently acquired in cities in the developing world with inadequate rodent control. Bartonella henselae causes cat scratch disease and bacillary angiomatosis. Its reservoir is the domestic cat. The vector for the cat population is the cat flea, but most human infection is acquired through cat scratches or cat bites. Borrelia recurrentis is a spirochete that causes louse-borne, relapsing fever. Humans are the only reservoir. Coxiella burnetii is the cause of Q fever. It is acquired by inhalation, and its reservoirs are sheep, goat, and cattle. Ehrlichia chaffeensis is a cause of human ehrlichiosis, which is characterized by leukopenia, thrombocytopenia, and anemia. It is transmitted by ticks, and deer are the reservoir.
A 26 year old female has pain and tenosynovitis of the wrists and ankles, and arthralgias of other joints. Two similar episodes have occurred before. She had her menstrual period during the previous week. Exam shows ulcerated lesions overlying the wrists and ankles. These symptoms are likely due to deficiency of
This patient has disseminated gonococcal infection. Gonococcal arthritis and tenosynovitis typically involve both the upper and lower extremities equally. Vesicular skin lesions are characteristic of disseminated gonococcal disease. Females are at particular risk of gonococcemia during menstruation, since sloughing of the endometrium allows access to the blood supply, necrotic tissue enhances the growth of Neisseria gonorrhoeae, and there is an alteration of the pH. Patients who have a C6-8 deficiency have both an increased risk of gonococcemia and a tendency to have multiple episodes. These patients are also at risk for bacteremia from Neisseria meningitides. Cl esterase inhibitor deficiency can occur as an autosomal dominant disorder or may be acquired. Patients have angioedema without urticaria. The syndrome is also associated with recurrent attacks of colic and episodes of laryngeal edema. Ciliary dysfunction is a marker of Kartagener syndrome (immotile cilia syndrome). The syndrome includes infertility, bronchiectasis, sinusitis, and situs inversus. It is an autosomal recessive disorder caused by abnormalities in the dynein arm of the cilia. Endothelial adhesion molecule deficiency, or beta-2 integrin deficiency, is characterized by failure of neutrophils to express CD18 integrins on their surface. Patients have impaired phagocyte adherence, aggregation, chemotaxis, and phagocytosis of C3b-coated particles. Clinically, there is delayed separation of the umbilical cord, sustained agranulocytosis, recur-rent infections of skin and mucosa, gingivitis, and periodontal disease. Eosinophil deficiency or eosinopenia occurs with stressors such as acute bacterial infection and following administration of glucocorticoids. There is no known adverse effect of eosinopenia
An 18-year-old girl developed pulmonary haemorrhage and acute renal failure requiring dialysis. Renal biopsy Crescentic glomerulonephritis. Which one of the following is most likely to be found in the blood?
This patient manifests a pulmonary renal syndrome which is most commonly due to an anti-neutrophil cytoplasmic antibody test (ANCA) positive vasculitis and less commonly due to Goodpasture´s syndrome (anti-glomerular basement membrane [GBM] antibodies). ANCA antibodies are of two types:
1. C-ANCA which correlates with anti-proteinase 3 antibodies (PR3) 2. P-ANCA which correlates with anti-myeloperoxidase antibodies. P- ANCA/MPO antibodies are highly sensitive and specific for rapidly progressive glomerulonephritis and haemorrhagic alveolar capillaritis. Anti-mitochondrial antibodies are found in primary biliary cirrhosis. Anti-centromere antibodies are found in CREST/scleroderma syndrome. ANA and anti-cardiolipin antibodies are found in systemic lupus erythematosus (SLE) which is not a cause of pulmonary renal syndrome. ASO is seen in rheumatic fever.
Regarding the genetics of bronchial asthma
Which one of the following factors will LEAST likely predispose an individual to the development of infective endocarditis?
Atherosclerosis is not considered a risk factor for the onset of infective endocarditis. Risk factors for infective endocarditis include: Marfan´s syndrome, black race, previous history of endocarditis, intravenous drug abuse, aortic stenosis, aortic regurgitation, mitral valve prolapse, mitral stenosis, cardiac myxomas, and other valve-related disorders
Chlamydiae are among the most common infectious agents in humans. All of the following are true concerning chlamydiae EXCEPT
Chlamydiae are obligate intracellular pathogens. Their extreme biosynthetic defects in intermediary metabolism and energy generation make them absolutely dependent on the host cell to grow and replicate. Depending on the species, chlamydiae only replicate intracellularly in the macrophage or in columnar epithelial cells. Infection of a previously exposed host results in an accelerated and intensified inflammatory response, with tissue destruction directly proportional to the inflammatory response. Host cell tropisms are correlated with the type of inflammation elicited. For example, Chlamydiae trachomatis lymphogranuloma venereum, which infects macrophages, produces a granulomatous inflammation characteristic of delayed hypersensitivity reactions.
An 80-year-old woman complains of a chronic cough. On examination, she is found to be wheezing. Which of the following conditions is NOT associated with wheezing?
A 7 day old premature infant, born at gestational age of 28 weeks dies after developing acute respiratory distress and acidosis, unresponsive to 100% oxygen administration. The autopsy reveals liver-like consolidation of the lungs. The histologic changes of the lungs are shown in the photomicrograph below. The underlying pathogenic mechanism of his condition is
The photomicrograph clearly demonstrates the histopathologic changes characteristic of hyaline membrane disease. This occurs in about 1-2% of term infants and 10-l5% of preterm infants. It is due to immaturity of the lungs and insufficient production of surfactant. Because of surfactant deficiency, the lungs retain the collapsed, atelectatic state of fetal life, and the infant develops acute respiratory distress a few hours to days after birth. Eosinophilic membrane-like structures lining the alveolar wall are the most characteristic changes of hyaline membrane disease. These pathologic features are similar to diffuse alveolar damage which causes adult respire to distress syndrome (ARDS) in adults. Surfactant is produced by type II pneumocytes.
Agenesis of kidneys with oligohydramnios (choice A) would result in a complex sequence of anomalies known as the Potter sequence. Oligohydramnios is caused by lack of urine production secondary to renal agenesis. In turn oligohydramnios results in compression of the fetus, with subsequent: development of flattened facies and foot deformities.
Congestive heart failure secondary to severe mitral stenosis (choice B) in an infant would present as dyspnea, respire to distress, and failure to thrive. This condition will also not have the histologic findings of hyaline membrane disease.
Pulmonary hypertension (choice D) is a consequence of several congenital cardiac abnormalities e.g. tetralogy of Fallot. Pulmonary hypertension leads to right ventricular hypertrophy and thickening of pulmonary arterioles.
Type I pneumocytes (choice E) do not produce surfactant. They are responsible for gas exchange.
Cysteine proteinases are inhibited by which of the following?
α2 macroglobulin inhibits the cysteine proteinase family of enzymes, while the other molecules do not. Tissue inhibitor of metalloproteinases (TIMP) is a specific inhibitor for the metalloproteinase enzymes. Aprotinin inhibits mast cell tryptase. αl proteinase inhibitor inhibits elastase. αl chymotrypsin inhibits cathepsin G.
Which of the following is NOT correct association between molecular genetic abnormalities and colon adenomas/carcinomas?
As currently understood, a series of genetic changes occur in concert to initially produce an adenoma that may subsequently progress to a carcinoma. The initial step probably involves a mutation of the K-ras proto-oncogene. The gene for familial adenomatous polyposis has been mapped to chromosome 5. Chromosome 17(p53 gene) and chromosome 18 (DDC gene) deletions are thought to be important in malignant transformation. Overexpression of the c-myc gene has been reported in colon cancers.
Workers in which of the following occupations are NOT at increased risk of lead poisoning?
It is estimated that 800,000 American workers have potentially significant exposure to lead. House painters who repair old houses are exposed to lead based paint. Demolition workers and others employed at firing ranges are exposed to intensive aerosol concentrations. Pottery makers are exposed to lead glazes. Storage battery workers are exposed to high lead concentrations in the batteries. Plastic manufacture workers are exposed to mercury.
In which of the following a Xanthomatous lesion is NOT seen?
Xanthomatous lesions are not found in association with hyperthyroidism. Hyperlipoproteinemias, diabetes mellitus, hypothyroidism, and pancreatitis can all be associated with xanthomas, which are characterized by lipid rich cells.
Infection with Treponema pallidum causes which one of the following?
Infection with Calymmatobacterium granulomatis causes painful beefy red granulomatous lesions. It is a sexually transmitted disease. H. ducreyi causes chancroid, which presents with a painful ulcer and secondary inguinal adenopathy. Treponema pallidum causes syphilis. Gardenerella spp. is a factor in bacterial vaginosis. Chlamydia trachomatis subtypes L1-L3 may cause lymphogranuloma venereum, which may produce a vesicular lesion followed by inguinal lymphadenitis and, eventually, bubo inguinale.
A 36 year old female has been unable to become pregnant for more than 3 years. According to her physician her fallopian tubes are scarred. She had been sexually promiscuous as a teenager. The most likely cause of her infertility is
Chlamydia trachomatis infection is a sexually transmitted disease which is the most common bacterial STD in the U.S. It is a major cause of pelvic inflammatory disease and subsequent tubal infertility. It can also cause ectopic pregnancies and pneumonia and conjunctivitis in newborns. Human papilloma virus infection is responsible for genital warts and a precursor for cervical cancer. Syphilis would cause primary infection by presenting a painless, external genital chancre at the site of exposure. Treponema pallidum is the responsible organism. Tuberculosis can also cause infertility by scarring the fallopian tubes, but it is not a sexually transmitted disease and this patient has had no prior exposure to TB. Mycobacterium tuberculosis can invade all organs of the body including the ovaries.
The risk of developing angiosarcoma is increased by exposure to which one of the following?
Occupational toxic substances such as polyvinyl chloride increase the risk of developing angiosarcoma. Infection by the liver fluke Clonorchis sinesis predisposes an individual to the development of hepatocellular carcinoma, as does anatoxin B1. Smoking increases the chance of developing a number of different neoplasms, such as renal cell carcinoma and transitional cell carcinoma of the bladder, but not angiosarcoma.
All of the following are a proven form of spread of the HIV virus, EXCEPT
All of the other choices are proven forms of transmission of this virus.
A 44 year old man presents with lower gastrointenstinal bleeding. On colonoscopy numerous polyps are seen. The patient also has increased pigmentation, especially over the inside of his cheek. The most likely diagnosis is
Peutz Jeghers syndrome consists of numerous polyps that are hamartomas (smooth muscle tumors) rather than adenomas and, therefore, have a lower malignancy potential. It is also associated with mucocutaneous pigmentation, especially of the buckle mucosa. Turcot syndrome refers to the association of colonic adenomas with CNS tumors. Gardner´s syndrome is familial polyposis and the presence of soft-tissue tumor and/or osteomas. Malignant potential is relatively high in familial polyposis. Juvenile polyposis presents with GI bleeding from polyps in the stomach, and the small and large intestines.
Multiple malformation syndromes does NOT include
SLE is not associated with multiple malformation syndromes, while all of the other conditions are.
A 46 year old diabetic female presents with severe prefrontal headache. Physician notices swelling of the orbit of the left eye. A sample of nasal discharge is taken on the left side and stained with silver stain that confirms zygomycosis. The organism reaches the brain via which one of the following?
Zygomycosis is caused by fungi of the genera Rhizopus, Absidia, or Mucor. Ketoacidotic diabetics are at risk for developing rhinocerebral infections following inhalation of the sporangiospores of these normally harmless bread molds. The cavernous sinuses are located on either side of the body of the sphenoid bone, and become a potential route of infection, because they receive blood both from the face (via the ophthalmic veins and sphenoparietal sinus) and some of the cerebral veins. The spread of infection, especially by Mucor sp, into the cavernous sinus, can produce either CNS infection or cavernous sinus thrombosis both of which are potentially fatal. The route from the face to the brain is not arterial. The superior sagittal sinus is located in the falx cerebri, and drains venous blood from the brain to other dural sinuses, from which it eventually drains into the jugular vein. The Zygomycophyta do not reach the brain by way of the superior sagittal sinus. The superior vena cava drains blood from the upper part of the body into the heart.
A 36 year old male sustains crush injuries to both legs. While hospitalized, he develops a severe infection in the legs and the tissues become hard to the touch and crepitant. Plain X-ray reveals formation of black pockets in the soft tissues. Feature of the most likely infectious agent that is responsible for the observed muscle pathology is
The disease described is gas gangrene, which is caused by the spore forming anaerobe Clostridium perfringens and other closely related species. These very dangerous bacteria particularly like to grow in necrotic vascular tissue, and they produce a variety of destructive enzymes and toxins. The toxin produced by C. perfringens that is most strongly associated with myonecrosis is the alpha toxin, which is a lecithinase, phospholipase C that lyses erythrocytes, platelets, leukocytes, and endothelial cells. This toxin mediates massive hemolysis, increased vascular permeability and bleeding, hepatotoxicity, bradycardia, and hypotension. In the laboratory, the action of the lecithinase can be observed in Nagler´s reaction, in which strains of C. perfringens will lyse the lecithin in egg yolk agar. Ability to degrade collagen is displayed by the kappa toxin of C. perfringens (a collagenase). It contributes to the necrotizing activity of the organism but is not the most important cause of the myonecrosis. Ability to degrade DNA is displayed by the toxin (a DNAse) of C. perfringens. It contributes to hemolysis and necrotizing activity of the organism but is not the most important cause of myonecrosis. Ability to degrade hyaluronic acid is expressed by the mu toxin (a hyaluronidase) of C. perfringens. Hyaluronidase is important in promoting the spread of the organism through the extracellular matrix but is not the most important mediator of myonecrosis. Ability to stimulate permeability is expressed by the epsilon toxin (a permease) of C. perfringens. It is produced as a protoxin, is activated by trypsin, and increases the permeability of the gastrointestinal tract. It is more important as a contributing cause to the development of necrotizing enteritis than to myonecrosis.
A 44 year old Nigerian presents with several months of weight loss, night sweats, sputum production, and occasional spitting up of blood. A protein injected into the skin of the forearm results in a 15-mm indurated swelling 48 hours later. Which phenomenon has most likely occurred? |
The tuberculin skin test is a delayed-type hypersensitivity test. This type IV hypersensitivity is a cell-mediated immune response involving CD4 T-helper 1 (TH1) lymphocytes activating and attracting macrophages into the area. This individual is TB-positive, and should now have cultures and a chest x-ray to confirm an active case of tuberculosis. An Arthus reaction has not occurred. The Arthus reaction is an example of a type III (immune complex) hypersensitivity involving antibody, antigen, and complement activation. Mast cell degranulation has not occurred. Mast cells are involved in type I hypersensitivities, along with IgE. Antibodies are not involved in the etiology of type IV hypersensitivity.
Neutrophil diapedesis has not occurred. Although neutrophils are always the first responders to any injury, by 48 hours they would no longer be the predominant cell in this area. It is not the case that TH2 cells have activated B cells because TH2 cells and B cells are involved in the production of the humoral immune response, and the tuberculin response is the classic demonstration of the function of the cell-mediated arm of the immune response.
A 10 year old child full thickness burn over 40% of his surface area develops pseudomonas infection over the burned areas of his body. He is treated with IV antibiotics but continues to show positive cultures. He becomes septic and develops fulminant pulmonary edema. A Swan-Ganz flow directed catheter placed into the pulmonary artery indicates that the pulmonary artery and pulmonary wedge pressures are both normal. Increase in which of the following is the likely cause of his pulmonary edema?
A major problem in patients with sepsis is a generalized increase in vascular permeability, which causes widespread edema. The increase in pulmonary microvascular permeability allows excess amounts of fluid and protein to leak into the interstitium of the lung, and from there into the alveoli. Pulmonary edema caused by increased microvascular permeability is characterized by an increase in interstitial fluid protein concentration, which causes interstitial colloid osmotic pressure to increase as well. Although the increased interstitial colloid osmotic pressure promotes the development of edema, the ultimate cause of the protein leakage into the interstitium is the high permeability of the microvasculature. The Interstitial fluid hydrostatic pressure is increased during pulmonary edema, which tends to decrease the amount of fluid extravasation from the vasculature. Lymph flow increases greatly as a consequence of edema; were it not for this increase in lymph flow, even greater amounts of fluid would collect in the lungs. The development of pulmonary edema during sepsis is often exacerbated by concomitant increases in pulmonary microvascular pressure; however, pulmonary artery pressure and wedge pressure were both normal in the patient, suggesting that Increased microvascular pressure did not contribute significantly to the development of edema. The pulmonary wedge pressure provides an estimate of left atrial pressure, and the pulmonary microvascular pressure is estimated as the average of the pulmonary artery pressure and the pulmonary wedge pressure
A radiologist notes the presence of line, radiographically dense crystals in the tissues of a knee joint. This patient most likely suffering from which one of the following types of arthropathy?
Radiographically dense calcium pyrophosphate dihydrate (CPPD) crystals are deposited in cartilage and joint soft tissues in pseudogout, which can involve the knees, wrists, elbows, shoulders, or ankles. The patient is asymptomatic because the deposition is primarily within cartilage, the condition is sometimes called chondrocalcinosis. In its more severe form, the joint involvement can clinically resemble rheumatoid arthritis. Neither gonococcal arthritis, osteoarthritis, nor rheumatoid arthritis is associated with crystal formation.
The monosodium urate crystals deposited in joints in gouty arthritis are not radiopaque.
A 67 year old man suffers MI due to thrombotic occlusion at the origin of the left circumflex artery. Cardiac catheterization reveals that he has a left dominant coronary circulation. Which areas of the heart have likely suffered ischemic necrosis?
A right dominant coronary circulation is present when the posterior descending branch originates from the right coronary artery (80% of individuals). On the contrary the posterior descending artery originates from the left circumflex artery in a left, dominant circulation (20% of individuals). The posterior descending branch gives blood to the posterior half of the interventricular septum. Occlusion of the left circumflex artery in a left dominant circulation will therefore lead to ischemic necrosis in the left ventricular wall and the posterior interventricular septum. The apex of the left ventricle is dependent on the anterior descending branch; thus, occlusion of the left circumflex does not affect this portion of the left ventricle. Infarction of the lateral (free) wall alone will result from occlusion of the circumflex in a right dominant circulation. An isolated infarction of the posterior interventricular septum arises from occlusion of the posterior descending branch. Isolated infarctions of the right ventricular wall are very are and would be caused by occlusion of branches of the right coronary artery.
A previously healthy girl aged 18 years old presents with fever, vomiting, and diarrhea. On exam, she is hypotensive and has an erythematous, red, sunburn-like skin rash. She is currently menstruating and has been using super absorbent tampons. Suspected diagnosis would be confirmed by which finding from a positive blood culture?
This is a multistep microbiology question that requires you to diagnose the illness, identify the microorganism, and remember its key feature. The first part should be easy: everything about this vignette suggests toxic shock syndrome. The organism in question is therefore Staphylococcus aureus, which is coagulase-positive. All the other choices are classic features of other important pathogenic microorganisms: Acid-fast organisms are associated with mycobacteria. (In addition, Nocardia species are partially acid fast.) EMB agar refers to a selective and differential medium used to isolate and identify enteric gram negative bacteria. Gram-positive bacteria will not grow on EMB agar because the addition of eosin inhibits their growth. Nonlactose fermenters will have colorless colonies; fermentation of this sugar will cause the colonies to appear pink or purple. Thayer Martin media can be used for pathogenic Neisseria species. This growth medium contains the antibiotic vancomycin, which kills gram-positive organisms such as Staphylococcus aureus. The Quellung reaction can be used to identify the capsule type of a microorganism. Encapsulated are mixed with specific antisera. If the antibodies recognize the microorganism´s capsule type, the capsule will swell
A 55 year old man has higher than normal language output and frequent paraphasic errors. Neurologic evaluation demonstrates that his comprehension of auditory and visual language is severely disturbed. He also has an inability to repeat language. He is suffering from which type of aphasia?
The question stem describes Wernicke, or receptive, aphasia. It is caused by a lesion in Wernicke´s area, which is located in the posterior part of the superior temporal gyms of the language dominant hemisphere.
Broca (expressive) aphasia is a disorder primarily of language output. Speech is slow and effortful, phrase length is short, and the patient tends to use only common nouns, verbs, and, occasionally, adjectives. Comprehension is relatively intact. Repetition is generally disturbed comparable to the amount of spontaneous output. It is produced by a lesion of Broca´s area, which is in the posterior part of the inferior frontal gyrus of the language dominant hemisphere. Conduction aphasia is produced by a lesion in the arcuate fasciculus, which is in the posterior inferior part of the parietal lobe of the language dominant hemisphere. This effectively disconnects Broca´s area from Wernicke´s area, resulting in the inability to repeat. Global aphasia occurs with the destruction of Broca´s area, Wernicke´s area, and the arcuate fasciculus, resulting in a combination of both Broca and Wernicke aphasias. Mixed transcortical aphasia results from damage to much of both Broca´s and Wernicke´s areas, but leaves the arcuate fasciculus intact. The patient has little spontaneous language output or comprehension but is still able to repeat.
A 41 year old Japanese male has abdominal discomfort characterized by sensations of fullness and mild pain after eating. He was previously diagnosed with a gastric ulcer, which has been refractory to traditional treatment. Biopsy reveals gastric carcinoma. The cancer is likely located in which region of the stomach?
Gastric carcinomas involve the antropyloric region in 50% to 60% of cases. Note that gastric carcinoma is the most common malignancy in Japan and typically presents with early satiety and pain after eating large meals.
Gastric carcinoma occurs at the apex of the cardia in 25% of cases. These patients generally present with dysphagia due to esophageal outlet obstruction. The greater curvature of the body is involved in only about 12% of cases of gastric carcinoma. However, when an ulcerating lesion does occur in this region, it is quite likely to be malignant. The antropyloric region is involved more often than the lesser curvature of the body, where lesions typically present with symptoms of peptic ulcer, not early satiety. Gastric carcinoma occurs much more commonly in the antropyloric region than near the gastroesophageal junction. Although the lesser curvature of the antropyloric region is a favored site, any nonhealing ulcer at any site in the stomach should be biopsied, as it may be cancer.
A boy aged 3 years is brought to the emergency department after the acute onset headache, vomiting nuchal rigidity, and impaired mental status. MRI reveals posterior foss tumor filling the 4th ventricle. Surgery is immediately started, and intraoperative consultation leads to a “frozen section” diagnosis of Medulloblastoma. The clinical feature that may be expected in this patient is which one of the following?
Any tumor “filling the 4th ventricle” blocks the circulation of cerebrospinal fluid (CSF). This blockage leads to increased intracranial pressure, which manifests with nausea vomiting headache nuchal rigidity mental status changes, and bilateral papilledema. Opening pressure should be elevated on lumbar puncture. If surgery is not performed promptly cerebellar tonsillar herniation and rapid death will ensue. In children medulloblastoma and ependymoma are the most frequent neoplasms presenting in this manner. Loss of peripheral vision is more commonly associated with pituitary tumors or craniopharyngiomas, which can impinge upon the optic chiasm. Muscular weakness is not typically associated with brain tumors but rather with demyelinating disorders neurodegenerative illnesses peripheral nerve lesions and systemic disease. Photophobia is generally associated with inflammation of the meninges such as in cases of meningitis or in subarachnoid hemorrhages which can produce a chemical meningitis. Tremors are associated with damage to the basal ganglia. Although a tumor in the 4th ventricle may presumably affect the cerebellum cerebellar lesions are more likely to produce ataxia and dysarthria than tremors.
Smear of a 35 year old female with long history of easy bruising shows only a few large, young platelets, while other cell lines are normal. Marrow studies show increased megakaryocytes. The likely diagnosis is
This clinical scenario is most consistent with idiopathic thrombocytopenic purpura, which is an immune mediated destruction of platelets by the spleen. The peripheral smear and marrow show the features described. Idiopathic thrombocytopenic purpura is distinguished from microangiopathic hemolytic anemia and thrombotic thrombocytopenic purpura by the absence of fragmented red cells. In both thrombasthenia and von Willebrand disease the platelet count will be normal, although platelet function is impaired.
This website uses cookies.